Real Analysis problem (easy), Triangle inequality

Click For Summary
The discussion focuses on proving the triangle inequality for a series of arbitrary real numbers, specifically that the absolute value of their sum is less than or equal to the sum of their absolute values. A participant shared their attempt in a PDF format, expressing uncertainty about notation and logic. They were advised to avoid flawed logical reasoning, particularly the misuse of implications in proofs. The correct approach involves proving the base case for n=1, assuming the inequality holds for n=k, and then demonstrating it for n=k+1 using the triangle inequality. This method effectively establishes the inequality for all natural numbers.
Levis2
Messages
43
Reaction score
0

Homework Statement



> a[1], a[2], a[3], .. , a[n] are arbitrary real numbers, prove that;

abs(sum(a, i = 1 .. n)) <= sum(abs(a), i = 1 .. n)

Homework Equations





The Attempt at a Solution



I have uploaded my attempt as a pdf file, since I'm not too familiar with the practice of writing it with correct notation on this great site :) i apologize. I am a 17-year old HS student, so i apologize if my attempt is way off or seems a bit thick .. This analysis is all new to me :)
 

Attachments

Physics news on Phys.org
You have the right approach, but be a little careful about your logic. You're using "OK, I want to prove statement A. But statement A implies statement B. Statement B is true, so statement statement A is true. " This is bad logic; let's say I want to prove 1=0. I add 5 to both sides to get 6=5 and I multiply both sides by 0 to get 0=0. This is true so 6=5! See what I mean? xP But it is kinda easy to do that--especially in these cases. xP

Anyway, in your case what you want to do is prove that it's true for n=1. (It's obviously true. o.o) Then, assume it's true for n=k. Add |a_{k+1}| to both sides and use the triangle inequality on the left. Hence, the inequality is true for all natural numbers.

Remember, induction is proving that IF n=k is true, THEN n=k+1 is true, which is what I did above. (Proving that n=1 is true, of course. xD)
 
Question: A clock's minute hand has length 4 and its hour hand has length 3. What is the distance between the tips at the moment when it is increasing most rapidly?(Putnam Exam Question) Answer: Making assumption that both the hands moves at constant angular velocities, the answer is ## \sqrt{7} .## But don't you think this assumption is somewhat doubtful and wrong?

Similar threads

  • · Replies 9 ·
Replies
9
Views
2K
  • · Replies 1 ·
Replies
1
Views
1K
  • · Replies 5 ·
Replies
5
Views
4K
  • · Replies 3 ·
Replies
3
Views
2K
  • · Replies 3 ·
Replies
3
Views
2K
  • · Replies 14 ·
Replies
14
Views
2K
Replies
9
Views
2K
Replies
7
Views
3K
  • · Replies 7 ·
Replies
7
Views
4K
  • · Replies 1 ·
Replies
1
Views
3K